Power Dissipation

Status
Not open for further replies.

Thread Starter

tjjam2003

Joined Apr 5, 2006
22
If a resistor of 350 ohms is connected across a source of 175 volts, with a current of 0.05 amps what is the power dissipation?? It should be 87.50W right??

What if the voltage is reduced to 105 volts, what is the power being dissipated by the resistor??

The answer in the book says : 31.5W

I'm not sure how they got that answer, how do I write it out to get the answer??


Thanks !!!
 

Papabravo

Joined Feb 24, 2006
21,158
Originally posted by tjjam2003@Apr 7 2006, 02:42 PM
If a resistor of 350 ohms is connected across a source of 175 volts, with a current of 0.05 amps what is the power dissipation?? It should be 87.50W right??

What if the voltage is reduced to 105 volts, what is the power being dissipated by the resistor??

The answer in the book says : 31.5W

I'm not sure how they got that answer, how do I write it out to get the answer??
Thanks !!!
[post=15951]Quoted post[/post]​
You must have been typing a bit fast. 175 volts on 350 ohms is 0.5 amperes, not 0.05 and yes that woul be 87.5 Watts. A substantial power resistor!

Twinkle twinkle little star
Power's equal I squared R
Rich (BB code):
I = E/R implies 105/350 = 0.3 Amperes

(0.3)*(0.3)*350 = 31.5 Watts

Also

P = E^2/R = ((105)*(105))/350 = 31.5
What kind of book gives you a problem, and does not show you how they got the answer? May be time for a new book!
 

Thread Starter

tjjam2003

Joined Apr 5, 2006
22
Originally posted by Papabravo@Apr 7 2006, 01:51 PM
You must have been typing a bit fast. 175 volts on 350 ohms is 0.5 amperes, not 0.05 and yes that woul be 87.5 Watts. A substantial power resistor!

Twinkle twinkle little star
Power's equal I squared R
Rich (BB code):
I = E/R implies 105/350 = 0.3 Amperes

(0.3)*(0.3)*350 = 31.5 Watts

Also

P = E^2/R = ((105)*(105))/350 = 31.5
What kind of book gives you a problem, and does not show you how they got the answer? May be time for a new book!
[post=15953]Quoted post[/post]​
Thanks...I'm sure I'll be back with another one.
 

Papabravo

Joined Feb 24, 2006
21,158
Originally posted by paultwang@Apr 7 2006, 02:53 PM
Edit: Papabravo has answered your question.
[post=15954]Quoted post[/post]​
Gee paultwang,

You could have left your answer up, after all you went to the trouble of creating it, and a second opinion or viewpoint never ever hurts.

cheers
 
Status
Not open for further replies.
Top